Line 5: Line 5:
 
[[Category: asan]]
 
[[Category: asan]]
 
[[Category: Exams]]
 
[[Category: Exams]]
 +
=Problem=
 +
The unit impulse response of an LTI system is the CT signal
 +
 +
<math> h(t)=e^{-t}u(t). \ </math>
 +
 +
What is the system's response to the input
 +
 +
<math>x(t)= u(t-1) ? \ </math>
 +
=Solution =
 
We are given the input to an LTI system along with the system's impulse response and told to find the output y(t).  Since the input and impulse response are given, we simply use convolution on x(t) and h(t) to find the system's output.
 
We are given the input to an LTI system along with the system's impulse response and told to find the output y(t).  Since the input and impulse response are given, we simply use convolution on x(t) and h(t) to find the system's output.
  

Latest revision as of 10:55, 30 January 2011

Problem

The unit impulse response of an LTI system is the CT signal

$ h(t)=e^{-t}u(t). \ $

What is the system's response to the input

$ x(t)= u(t-1) ? \ $

Solution

We are given the input to an LTI system along with the system's impulse response and told to find the output y(t). Since the input and impulse response are given, we simply use convolution on x(t) and h(t) to find the system's output.

$ y(t) = h(t) * x(t) = \int_{-\infty}^\infty h(\tau)x(t-\tau)d\tau $


Plugging in the given x(t) and h(t) values results in:

$ \begin{align} y(t) & = \int_{-\infty}^\infty e^{-\tau}u(\tau)u(t-\tau-1)d\tau \\ & = \int_0^\infty e^{-\tau}u(t-\tau-1)d\tau \\ & = \int_0^{t-1} e^{-\tau}d\tau \\ & = 1-e^{-(t-1)}\, \mbox{ for } t > 1 \end{align} $


Since x(t) = 0 when t < 1:

$ y(t) = 0\, \mbox{ for } t < 1 $


$ \therefore y(t) = \begin{cases} 1-e^{-(t-1)}, & \mbox{if }t\mbox{ is} > 1 \\ 0, & \mbox{if }t\mbox{ is} < 1 \end{cases} $

Alternative Solutions

Problem 5 - Alternate Solution

Problem 5 - Graphical Solution

Alumni Liaison

Ph.D. on Applied Mathematics in Aug 2007. Involved on applications of image super-resolution to electron microscopy

Francisco Blanco-Silva